Stay ahead of learning milestones! Enroll in a class over the summer!

G
Topic
First Poster
Last Poster
Geometry :3c
popop614   4
N 7 minutes ago by goaoat
Source: MINE :<
Quadrilateral $ABCD$ has an incenter $I$ Suppose $AB > BC$. Let $M$ be the midpoint of $AC$. Suppose that $MI \perp BI$. $DI$ meets $(BDM)$ again at point $T$. Let points $P$ and $Q$ be such that $T$ is the midpoint of $MP$ and $I$ is the midpoint of $MQ$. Point $S$ lies on the plane such that $AMSQ$ is a parallelogram, and suppose the angle bisectors of $MCQ$ and $MSQ$ concur on $IM$.

The angle bisectors of $\angle PAQ$ and $\angle PCQ$ meet $PQ$ at $X$ and $Y$. Prove that $PX = QY$.
4 replies
popop614
Yesterday at 12:19 AM
goaoat
7 minutes ago
No more topics!
ABCD cyclic --> incenters of BCD, etc. form rectangle
Arne   12
N Oct 23, 2023 by ismayilzadei1387
Source: Kedlaya; also: APMO 1996, Problem 3
If $ABCD$ is a cyclic quadrilateral, then prove that the incenters of the triangles $ABC$, $BCD$, $CDA$, $DAB$ are the vertices of a rectangle.
12 replies
Arne
Jan 23, 2005
ismayilzadei1387
Oct 23, 2023
ABCD cyclic --> incenters of BCD, etc. form rectangle
G H J
Source: Kedlaya; also: APMO 1996, Problem 3
The post below has been deleted. Click to close.
This post has been deleted. Click here to see post.
Arne
3660 posts
#1 • 2 Y
Y by Adventure10, Mango247
If $ABCD$ is a cyclic quadrilateral, then prove that the incenters of the triangles $ABC$, $BCD$, $CDA$, $DAB$ are the vertices of a rectangle.
Z K Y
The post below has been deleted. Click to close.
This post has been deleted. Click here to see post.
kueh
392 posts
#2 • 6 Y
Y by Pluto1708, Adventure10, anonman, Mango247, Stuffybear, and 1 other user
By angle chasing.

You can show that the incentres of ABC (K), BCD (L), B and C are cyclic because angle CLB = 180 - DCB/2 - DBC/2 = 90 + BDC/2, and similarly, angle CKB = 90 + BAC/2. But BAC = BDC. Thus CLB = CKB and so B,C,K,L are concyclic. Also if we call the incentre of ADC (M), D,C,L,M is cyclic. angle KLC = 180 - angle LBC, angle MLC = 180 - MDC, thus angle KLM = angle KBC + angle MDC = (ADC+ABC)/2 = 90, which is what we want.

PS: Can someone give me a crashcourse on latex? or point to me where?
Z K Y
The post below has been deleted. Click to close.
This post has been deleted. Click here to see post.
darij grinberg
6555 posts
#3 • 2 Y
Y by Adventure10, Mango247
It can be proved by complex numbers... See also

Paul Yiu, Notes on Euclidean Geometry, paragraph 10.2.6 (page 154);
Barry Wolk, Hyacinthos message #8154,
Alexey A.Zaslavsky, Hyacinthos message #8178.

darij
This post has been edited 1 time. Last edited by darij grinberg, Jun 15, 2006, 3:54 PM
Z K Y
The post below has been deleted. Click to close.
This post has been deleted. Click here to see post.
pavel kozlov
613 posts
#4 • 1 Y
Y by Adventure10
There is the converse of this fact (from Russian Summer TST 2005):
Let $ABCD$ be a quadrilateral. The incentres of the triangles $ABC, BCD, CDA, DAB$ form a rectangle. Proove, that $ABCD$ is cyclic.
It's very interesting and rather difficult problem.
Z K Y
The post below has been deleted. Click to close.
This post has been deleted. Click here to see post.
jayme
9775 posts
#5 • 3 Y
Y by govind7701, Adventure10, Mango247
Dear Mathlinkers,

http://jl.ayme.pagesperso-orange.fr/Docs/Le%20rectangle%20de%20Ryokan%20Maruyama.pdf

Sincerely
Jean-Louis
Z K Y
The post below has been deleted. Click to close.
This post has been deleted. Click here to see post.
purplish
299 posts
#6 • 3 Y
Y by Pluto1708, Adventure10, Mango247
We use complex numbers. Plot the points on the unit circle. Then we can choose $a,b,c,d$ such that $A=a^2,B=b^2,C=c^2,D=d^2$, and \[I_{ABC}=-bc-ca-ab,I_{BCD}=-cd-db-bc,I_{CDA}=da+ac-cd,I_{DAB}=-ab+bd+da.\]Then we can compute \[I_{ABC}+I_{CDA}=-ab-bc-cd+da=I_{BCD}+I_{DAB}\]and
\begin{align*}
\left|I_{ABC}-I_{CDA}\right|^2&=\left|-bc-2ca-ab-da+cd\right|^2=\frac{1}{abcd}\left(-bc-2ca-ab-da+cd\right)\left(-ad-2bd-cd-bc+ab\right)\\
&=\frac{1}{abcd}\left(-cd-2db-bc-da+ab\right)\left(-ab-2ac-ad-bc+cd\right)=\left|-cd-2db-bc-da+ab\right|^2=\left|I_{BCD}-I_{DAB}\right|^2
\end{align*}so the incenters form a rectangle.
Z K Y
The post below has been deleted. Click to close.
This post has been deleted. Click here to see post.
zuss77
520 posts
#7 • 1 Y
Y by yellowhanoi
Labels are on the diagram.
By Fact 5 $AMNB$ is cyclic. And so is $AMQD$.
$\angle NMQ=\angle B/2 +\angle D/2=90^\circ$. And similarly the others.
Attachments:
Z K Y
The post below has been deleted. Click to close.
This post has been deleted. Click here to see post.
denery
180 posts
#8
Y by
i only could find those right angles on the rectangle by drawing external tangents
and since it is a quadrilateral
it must be a square or rectangle
and by length chasing interestingly we could say that that square/rectangle is definitely composed of rectangles
This post has been edited 1 time. Last edited by denery, Dec 15, 2020, 6:21 AM
Z K Y
The post below has been deleted. Click to close.
This post has been deleted. Click here to see post.
primesarespecial
364 posts
#9
Y by
We use complex numbers on this one.
We just easily prove this a parallelogram and then just show perpendicularity once or prove diagonals equal in length. :blush:
Z K Y
The post below has been deleted. Click to close.
This post has been deleted. Click here to see post.
sunken rock
4379 posts
#10
Y by
My solution at https://artofproblemsolving.com/community/c6t48f6h24483_abcd_cyclic_gt_incenters_of_bcd_etc_form_rectangle
I hope you will enjoy it!

Best regards,
sunken rock
Z K Y
The post below has been deleted. Click to close.
This post has been deleted. Click here to see post.
archimedes26
612 posts
#11
Y by
https://groups.io/g/euclid/topic/81006484#1476
Z K Y
The post below has been deleted. Click to close.
This post has been deleted. Click here to see post.
archimedes26
612 posts
#12
Y by
Another property.
https://geometry-diary.blogspot.com/2020/11/1521.html
Z K Y
The post below has been deleted. Click to close.
This post has been deleted. Click here to see post.
ismayilzadei1387
219 posts
#13 • 1 Y
Y by FriIzi
Also China 1986 TsT
Z K Y
N Quick Reply
G
H
=
a